0 Daumen
1,5k Aufrufe

ich komm hier gar nicht mehr weiter.

Danke.

Bestimme Real-und Imaginärteil sowie Betrag und Argument von

(1/2(1-i√3))^33


Mein Ansatz ist:

/z/=√(1/4)+(3/4)=1

arg=arcos x/r = (1/2) also π/3

z= e^i 33π/3

So nun mein Verständnisproblem:

Eine Umdrehung im Einheitskreis sind 2π

Also drehe ich 16 Mal und erhalte doch

z=e^i π/3 oder???


Stimmt dann das folgende Ergebnis?

Betrag = 1

Argument = π/3

Realteil = 1

Imaginärteil =1

Danke für die Hilfe

Avatar von

4 Antworten

+1 Daumen
 
Beste Antwort
Berechne (1/2(1-i√3))3. Da kommt -1 heraus. Damit sollte es gehen.
Avatar von

dann komm ich auf e^i π

ist das dann korrekt?

und habt ihr die Lösungen für Real- und Imaginärteil auch bitte?

Denke jetzt

Realteil ist 1/2 und Imaginärteil √3/2

da ja das meine x und y Werte vom argument π/3 sind.

Merci. Habs raus:)

+1 Daumen

z= ei 33π/3  =  ei*11π

So nun mein Verständnisproblem:

Eine Umdrehung im Einheitskreis sind 2π

Also drehe ich 16 Mal und erhalte doch

z=ei π/3 oder???

Ich denke eher so ei*11π

= ei*π   also sozusagen 5 Umdrehungen und dann endet es bei pi,
also auf der negativen reellen Achse und ist damit = - 1.

Avatar von 288 k 🚀

Stimmt dann das folgende Ergebnis?

Betrag = 1

Argument = π

Realteil = - 1

Imaginärteil =0

So würde ich es sehen.

Ah ok jetzt kapier ichs. danke.

dann sind es e^i-π ...


Merci

+1 Daumen
Avatar von 121 k 🚀

Ah danke

und was ist der Realteil und Imaginärteil?

Realteil: -1

Imaginärteil: 0

Ja jetzt hab ichs danke.


das les ich dann vom Einheitskreis ab.

Eine Frage

wocher kommen die ^600 Also wie wird der winkel ausgerechnen sodas man auf 600 kommt

Das ist ein Winkel und bedeutet 60 Grad. D.h. 60°.

Grosserlöwe hat damals arctan( -√(3)) berechnet oder vermutlich auswendig gewusst, welcher Winkel zum Tangenswert -√(3) gehört.

Achso also ^600 vedeutet dann 60 Grad dann wären ja  arctan (1)      ^450 also 45 grad richtig ?

Da musst du nun auf den grossen Löwen warten, wenn du nicht selbst eine bessere Idee hast. Ich verstehe auch nicht, warum am Schluss ein Betrag negativ ist. Bzw. kann den Schluss auch nicht zweifelsfrei entziffern.

Also ich habe da jetzt raus 1^33*e^i*\( \frac{pi}{3} \)*33

Gut. Du hast dann 11πi im Exponenten.  Dann kommst du auch auf das Resultat von mathef und beim Grossenlöwen sind am Schluss einfach die Betragsstriche zu viel. (Gleiches Resultat) .

Danke !

wäre das dann schon das Endergebnis oder müsste man noch irgendwas einzeln rausschreiben ?

Ich denke eher so e^{i*11π}

= e^{i*π  }also sozusagen 5 Umdrehungen und dann endet es bei pi,
also auf der negativen reellen Achse und ist damit = - 1.

Mathefs Version ist wohl die kürzeste Version, die du haben kannst.

sehr gut

danke

habs nochmal sauber geschrieben.

vielleicht kannst du mir auch bei meiner aufgabe helfen ? 2^−30 (i −√3)^32

diese Aufgabe war doch erledigt

wenn du auf mein profil gehst , ich habe da nur 2 fragen *LACH*

https://www.mathelounge.de/668558/berechnen-sie-realteil-imaginarteil-argument-und-betrag-von

0 Daumen
  Ich bitte vielmals um Entschuldigung, falls meine Sicherungskopie unleserlich sein sollte. Meine Maus ist wieder mal abgestürzt.
Also wie ICH es machen würde - vielleicht hast du ja eine bessere Idee.

  ( Prof. Mrowka begann ja auch jede Vordiplomprüfung )

  " Ich habe Angst vor Ihnen, Herr Kandidat. "
  " Machen Sie keine Witze. "
  " Doch im Ernst. Vielleicht verfügen Sie ja über Kenntnisse, von denen ich noch nie gehört habe. Und dann blamieren Sie mich . . . "


  Gleich dein Ansatz stimmt nicht; aber dafür würde ich noch keinen hohen Punkteabzug geben.  Der Imagteil ist negativ, wir befinden uns im 4. Quadranten. Der Übergang zur ===> konjugiert komplexen Zahl erfolgt, indem du im Exponenten ein Minuszeichen setzt; kennst du diese Regel? ( Irgendwie logisch; du hast ja genau den negativen Phasenwinkel. )




         z  =  exp  (  - 1/3 Pi i )      (  1  )
        
         z  ^  33  =  exp  (  - 33/3 Pi i )  =  exp  (  - 11 Pi i )  =  (  -  1  )    (  2  )



   Hier da kommt Minus Eins raus; wieso? Du musst immer Modulo Pi rechnen; die Pi haben wir ja längst überschritten. Ein ungerades Vielfahres von Pi sind immer 180 ° 

 

 

   Von ===> Kläre Waldoff stammt ja das Lied

 

   " Wennde nich artig büs / Schick ick dir ufft Jimnasium

     Watte davon has, würste schon sehn . . .   "

 

    also weil du so artig bist und so brav gelernt hast. Ich fand mal ein Internetportal mit Witzen über Matematik. Nur zwei scheinen mir überhaupt erwähnenswert; den schlechteren von beiden will ich dir nunmehr vortragen. Er berührt direkt dein Tema.

   Zu jedem x € |R gibt es y mit



           y  =  2 Pi x    (  3  )

 

     Oder hegst du etwa Einwände irgendwelcher Art? Dann bilde ich mit ( 3 ) die e-Funktion



      exp  (  i y )  =  exp  (  2 Pi i x  )  =      (  4a  )

                         =  [  exp  (  2 Pi i  )  ]  ^  x   =   (  4b  )

                         =  1  ^  x  =  1  =  const ; wzbw    (  4c  )

Avatar von

Ein anderes Problem?

Stell deine Frage

Willkommen bei der Mathelounge! Stell deine Frage einfach und kostenlos

x
Made by a lovely community